Difference between revisions of "1987 AJHSME Problems/Problem 9"
5849206328x (talk | contribs) (New page: ==Problem== When finding the sum <math>\frac{1}{2}+\frac{1}{3}+\frac{1}{4}+\frac{1}{5}+\frac{1}{6}+\frac{1}{7}</math>, the least common denominator used is <math>\text{(A)}\ 120 \qquad \...) |
5849206328x (talk | contribs) m (→Solution) |
||
Line 7: | Line 7: | ||
==Solution== | ==Solution== | ||
− | We want <math>\text{LCM}(2,3,4,5,6,7)</math>, which is <math> | + | We want <math>\text{LCM}(2,3,4,5,6,7)</math>, which is <math>420</math>, or choice <math>\boxed{\text{C}}</math>. See [[Least common multiple|here]] if you don't know how to evaluate LCMs. |
==See Also== | ==See Also== | ||
[[1987 AJHSME Problems]] | [[1987 AJHSME Problems]] |
Revision as of 21:39, 14 February 2009
Problem
When finding the sum , the least common denominator used is
Solution
We want , which is , or choice . See here if you don't know how to evaluate LCMs.